lunes, 3 de enero de 2011

Problema del Día (3 de Ene)

Sea $n \geq 3$ un entero. Sean $t_1,t_2,\cdots,t_n$ reales positivos tales que:

\[n^{2}+1 >\left( t_{1}+t_{2}+...+t_{n}\right)\left(\frac{1}{t_{1}}+\frac{1}{t_{2}}+...+\frac{1}{t_{n}}\right)\]


Prueba que $t_i,t_j,t_k$ son los lados de un triangulo para toda $i,j,k$ con $ 1\leq i \textless j \textless k\leq n $.

1 comentario: